Please confirm topic selection

Are you sure you want to trigger topic in your Anconeus AI algorithm?

Please confirm action

You are done for today with this topic.

Would you like to start learning session with this topic items scheduled for future?

Review Question - QID 214776

In scope icon M 6 A
QID 214776 (Type "214776" in App Search)
A 42-year-old woman presents to the clinic with complaints of premature menopause. She describes episodes of “hot flashes” with associated heart palpitations for the past 3 months. However, she has continued to have regular menstrual periods during this time. She also endorses fatigue, increased appetite, and an unintentional 5-lb. weight loss. Past medical history includes diabetes that is currently diet-controlled and an upper respiratory infection 3 weeks ago that has resolved. Her temperature is 99.6°F (37.6°C), blood pressure is 145/90 mmHg, pulse is 115/min, and respirations are 21/min. A physical examination demonstrates tachycardia and a nontender palpable nodule at the neck. Laboratory studies are shown below, and a radioactive iodine uptake test (RAIU) is shown in Figure A.

Hemoglobin: 14 g/dL
Hematocrit: 39%
Leukocyte count: 10,900/mm^3 with normal differential
Platelet count: 210,000/mm^3

Serum:
Na+: 138 mEq/L
Cl-: 3.7 mEq/L
K+: 4.2 mEq/L
HCO3-: 25 mEq/L
BUN: 11 mg/dL
Glucose:134 mg/dL
Creatinine: 0.9 mg/dL
Thyroid-stimulating hormone (TSH): 0.1 uU/mL
Free T4: 37 ug/dL
Anti-thyroid peroxidase antibody (anti-TPO): 10 IU/mL (Normal: < 35 IU/mL)

Which of the following is the most likely explanation for this patient’s findings?

  • A
  • A